LSAT and Law School Admissions Forum

Get expert LSAT preparation and law school admissions advice from PowerScore Test Preparation.

 LSAT2018
  • Posts: 242
  • Joined: Jan 10, 2018
|
#44605
Not Band Together → Proposal Approved → Build Appartment → Attract New Residents → Overcrowding → New Roads → Substantial Tax Increases
(A) Mistaken Negation
(B) Chain
(C) Mistaken Negation
(D) Mistaken Negation
(E) Mistaken Negation

Apart from answer (B) I found all the other choices to be Mistaken Negation.
But to clarify some parts in the stimulus, I noticed that there were some causal reasoning within the conditional reasoning chain such as in the third sentence 'the increased population would probably result in overcrowded schools and would certainly result in roads so congested that new roads would be built'. How is this valid? Is it correct that conditional reasoning statements be converted to causal statements (but not the other way around)?
 Shannon Parker
PowerScore Staff
  • PowerScore Staff
  • Posts: 147
  • Joined: Jun 08, 2016
|
#44687
Conditional statements and causal statements are distinct but often overlap. For instance is one variable causes another variable to occur 100% of the time, there will be a conditional relationship. i.e., if A causes B 100% of the time we know that A--->B.

But conditional relationships in no way create causation (i.e. just because B happens 100% of the time A happens, does not mean that A causes B).

Hope this helps.
Shannon
 LSAT2018
  • Posts: 242
  • Joined: Jan 10, 2018
|
#44748
But if conditional statements can be applied when there is 100 percent causal relationship (if A causes B 100 percent of the time we know that A → B) then the conditional statement is only:
Not Band Together → Proposal Approved → Build Appartment → Attract New Residents → New Roads → Substantial Tax Increases

And the part about overcrowding cannot be placed in the conditional since it doesn't indicate a 100 percent relationship?(according to the stimulus, 'the increased population would probably result in overcrowded schools')
 Shannon Parker
PowerScore Staff
  • PowerScore Staff
  • Posts: 147
  • Joined: Jun 08, 2016
|
#44877
LSAT2018,

You are correct, but be mindful that this does not effect the reasoning. The stimulus states that Neither new roads, or schools can be built without a substantial tax increase. Thus the buildings leads to new residents, which leads to new roads, which leads to a tax increase.

Keep it up.
Shannon
 S2KMo
  • Posts: 12
  • Joined: Jun 10, 2018
|
#46707
Hello,

I understand the chain of conditional statements, however, the only thing that I thought had an impact on the chain (which clearly didn't) was in the third sentence it says: ...and the increased population would probably result in overcrowded schools. Did the fact that this said probably, not change the relationship, because it's not 100% certain that it does result in overcrowded schools, which is partially sufficient for the substantial tax increase?
 James Finch
PowerScore Staff
  • PowerScore Staff
  • Posts: 943
  • Joined: Sep 06, 2017
|
#46719
Hi Mo,

Conditional logic requires 100% certainty to function, because whenever a sufficient condition is true then a necessary condition must also be true. "Probably" introduces doubt, and while we could fork the diagram to account for the doubt, the question stem itself gives us a hint as to the level of certainty we need for the correct answer choice.

Here, we are asked what can be "properly concluded" from the passage, which means known to 100% certainty. For Prephrasing purposes, when there are two chains, one certain and one merely probable, the one that corresponds with the level of certainty given in the question stem is much more likely to be the correct answer choice. Answer choice (B) correctly describes the definite link between building apartments and substantial tax increases, making it correct.

Hope this helps!
 dandelionsroar
  • Posts: 27
  • Joined: Oct 18, 2018
|
#60440
Hello,

Is the first answer choice a mistaken negation or a mistaken reversal? Also regarding Evans tip (to look at the ends of the logic chain for the correct answer choice) is that usually the case? So when narrowing down the answer choices I should start off by looking for a contrapositive that includes a condition that either starts or ends the logical chain?

Thanks!!:)
 Robert Carroll
PowerScore Staff
  • PowerScore Staff
  • Posts: 1819
  • Joined: Dec 06, 2013
|
#60690
dandelion,

In order to answer your question about answer choice (A), I'm going to go over the conditionals in the stimulus a bit. This will rehash some discussion in this thread already.

The first sentence is a statement using the word "unless". Because of that, we can employ the Unless Equation and diagram it as follows:

rezoning proposal approved :arrow: residents band together

Looking at the rest of the stimulus, I see nowhere else where it discusses the residents banding together. So when answer choice (A) says "if they band together" as its sufficient condition, I can see that could only be inferred from the first sentence. But it can't be inferred from that. The necessary condition of the first sentence is "residents band together." If I took the contrapositive of the first sentence, I would get:

residents band together :arrow: rezoning proposal approved

"residents band together" is still not sufficient - its negation is! So answer choice (A) can't be inferred from the stimulus.

What answer choice (A) is doing is, as you suspected, taking a Mistaken Reversal of the first sentence, and then chaining that mistake with the rest of the stimulus. Because that depends on an initial mistake (reversing the first sentence), the whole thing will be wrong.

In short, it's a Mistaken Reversal.

Now, the contrapositive of a Mistaken Reversal is a Mistaken Negation, so really each mistake is a different way of saying the same (erroneous) thing. So I wouldn't get too worried about which mistake is being made - that could only matter in a Parallel Reasoning question, not a Must Be True.

As to your second question, what Jonathan Evans said about Must Be True questions and long conditional chains is spot on. I tell my students the same thing - if a stimulus has multiple conditionals that can be validly combined and the question is a Must Be True, the answer usually involves the a large part of the chain, often the entire thing! So it's good to look for long chains in answer choices for MBT questions. A contrapositive of that chain is equally valid as an answer!

Robert Carroll
 kalifaingold
  • Posts: 11
  • Joined: Jan 14, 2020
|
#73384
Emily Haney-Caron wrote:Hi Amna,

The reasoning here is basically one long string. Here's what I have:
Don't band together :arrow: proposal approved :arrow: apartments built :arrow: new residents :arrow: overcrowded schools and roads :arrow: new roads built :arrow: residential tax increases
This is somewhat unusual, but basically every single piece of information builds on and connects to the piece of information before it. Does that help?
Is this not the contrapositive of the stimulus? Using the unless equation you get the conditional relationship: "No zoning approval- residents ban together" correct?
Can you please diagram the full chain as well as the contrapositive of the chain
 kalifaingold
  • Posts: 11
  • Joined: Jan 14, 2020
|
#73385
Jonathan Evans wrote:"Unless the residents of Glen Hills band together, the proposal to rezone that city will be approved."

~GHBT :arrow: PRZA

CP: ~PRZA :arrow: GHBT

"If it is, the city will be able to build the water and sewer systems that developers need in order to construct apartment houses there."

PRZA :arrow: ABW&SS

CP: ~ABW&SS :arrow: ~PRZA

"These buildings would attract new residents, and the increased population would probably result in overcrowded schools and would certainly result in roads so congested that new roads would be built."

ABW&SS :arrow: BNR

CP: ~BNR :arrow: ~ABW&SS

"Neither new roads nor additional schools could be built without substantial tax increases for the residents of Glen Hills."

BNR :arrow: STI

CP: ~STI :arrow: ~BNR

"Ultimately this growth might even destroy the rural atmosphere that makes Glen Hills so attractive."

This last statement is superfluous.

Zero in on the definite relationships, the definite sufficient and necessary conditions. Attempt to excise the "might be"/"could be" fluff from a string of conditionals. Do not overthink conditionals and create unnecessary intermediate steps.

Also key, expect to find a credited answer choice in these string conditional Must Be True questions that addresses at least one end of the terminal chain of reasoning, be it at the beginning or the end.
Hi, this is very helpful. But nowhere has anyone explained to me how to get the contrapositive of THE WHOLE CHAIN. How would I go about doing this? Like how do I find the contrapositive of the chain as a whole in order to see that answer B is correct?

Get the most out of your LSAT Prep Plus subscription.

Analyze and track your performance with our Testing and Analytics Package.